NRNP 6665-01 Week 11 Final Exam 2022/2023 (100% Correct Answers & Explanations)

Question 18:
Christine is a 9-year-old female who presents for care after having been placed in the local foster care system. She has been in and out of foster care for the last 4 years after her parents were killed in an automobile accident. Christine has been placed in a variety of homes and residential care facilities. The PMHNP recognizes that Christine is at high risk for:
A. Dissociative disorders

B. Post-traumatic stress disorder

C. Impulse-control disorder

D. Attachment disorder

Question 7:
Caylee is a 5-year-old girl who is referred for evaluation by child protective services. She was recently removed from her biological family and placed in foster care, as her home environment was reportedly unsafe due to conditions of extreme neglect. Her foster mother reports that Caylee is very quiet and withdrawn and always appears sad and disinterested in her surroundings; however, she becomes very irritable when anything unexpected or unplanned occurs. The foster mother became very concerned when it appeared that Caylee was hallucinating. The PMHNP considers that:
A. Caylee is at high risk for suicide and precautions should be taken

B. The hallucinations are consistent with brief psychotic disorder or schizophrenia

C. The history and reported symptoms are typical of depressive disorder in young children

D. This is a common situation when prepubertal children are removed from the biological parents regardless of how dysfunctional they are

Question 2:
Debi is a 15-year-old girl who is currently being treated for depression. Her parents have been very proactive and involved in her care, and Debi has achieved remission 2 months after beginning treatment with a combination of pharmacotherapy and cognitive behavioral therapy. While counseling Debi’s parents about important issues in management, the PMHNP advises that:
A. There is a > 50% likelihood that Debi’s younger sibling will develop depressive symptoms

B. The mean length of major depressive episode in adolescents is 4 months

C. 20 to 40% of adolescents who have major depressive disorder will develop bipolar I within 5 years

D. Adolescent-onset depression typically need long-term pharmacologic management to prevent relapses

Agomelatine
Brand: Valdoxan
-Melatonin multimodal (Mel-MM)
-Agonist at melatonergic 1 and melatonergic 2 receptors
-Antagonist at 5HT2C receptors
-Not FDA approved: Rx for Depression, Generalized anxiety disorder
-Initial 25 mg/day at bedtime; after 2 weeks can increase to 50 mg/day at bedtime

Amitriptyline (Elavil)
Tricyclic antidepressant.

Aripiprazole (Abilify)
Treatment for depression. Atypical antipsychotic. “Dopamine stabilizer”. Dopamine receptor antagonist in high concentration and also stimulates increase of dopamine in low concentrations. Side effects: insomnia, akathisia.

Brexpiprazole (Rexulti)
Treatment for depression. Atypical antipsychotic

Bupropion (Wellbutrin)
Antidepressant and smoking cessation aid, It can treat depression and help people quit smoking. It can also prevent depression caused by seasonal affective disorder (SAD).

Citalopram (Celexa)
Antidepressant, SSRI: 20-40 mg qd.

Clomipramine (Anafranil)
-Serotonin reuptake inhibitor (S-RI)
-Tricyclic antidepressant (TCA)
-Parent drug is a potent serotonin reuptake inhibitor
Active metabolite is a potent norepinephrine/noradrenaline reuptake inhibitor
-Increases serotonergic neurotransmission by blocking the serotonin reuptake pump (transporter), -Desensitization of serotonin receptors, especially serotonin 1A receptors
-Increases noradrenergic neurotransmission by blocking the norepinephrine reuptake pump (transporter), -Desensitization of beta adrenergic receptors
-Dopamine is inactivated by norepinephrine reuptake in the frontal cortex
-Lacks dopamine transporters
-Increases dopamine neurotransmission in this part of the brain
**FDA Approved for Pediatrics in tx Obsessive-compulsive disorder (OCD) (ages 10 and older)
*Off-Label for Pediatric Use: Depression, Severe and treatment-resistant, depression, Cataplexy syndrome, Anxiety, Insomnia,
Neuropathic pain/chronic pain
-Full therapeutic benefits may take 2-8 weeks
-Dosing: in Peds/Adolescents/Adults:
100-250mg/day

Cyamemazine (Tercian)
Treatment for depression.

-Known as cyamepromazine

-Typical antipsychotic drug of the phenothiazine class.

-Treatment for schizophrenia

and psychosis-associated

anxiety

-Behaves like an atypical

antipsychotic, due to its

potent anxiolytic effects (5-HT2C) and lack of extrapyramidal side effects (5-HT2A).

-Conventional antipsychotic (neuroleptic, phenothiazine, dopamine 2 antagonist, serotonin dopamine antagonist)

-Commonly Prescribed for (not FDA approved):

-Schizophrenia

-Anxiety associated with psychosis (short-term)

-Anxiety associated with nonpsychotic disorders, including mood disorders and personality disorders (short-term)

-Severe depression

-Bipolar disorder

-Other psychotic disorders

-Acute agitation/aggression (injection)

-Benzodiazepine withdrawal

Desiprimine (Norpramine)
Treats depression
Brand: Norpramin
-TCA
-Norepinephrine noradrenaline reuptake inhibitor.
-FDA approved for treating depression.
-Off-label: Anxiety, Insomnia,
Neuropathic pain/chronic pain, Treatment-resistant depression.
-More potent inhibitor of norepinephrine reuptake pump than serotonin reuptake pump (serotonin transporter).
-May have immediate effects in treating insomnia or anxiety.
-If it is not working within 6-8 weeks for depression, it may require a dosage increase or it may not work at all
-100-200 mg/day (for depression)
50-150 mg/day (for chronic pain).

Desvenlafaxine (Pristiq)
Dual serotonin and norepinephrine reuptake inhibitor-often classified as an antidepressant.
-FDA approved to tx MDD.
-Off-label: Vasomotor sx’s, fibromyalgia, GAD, Social Anx d/o, panic d/o, PTSD, PMDD

-Dopamine is inactivated by norepinephrine reuptake in frontal cortex (which lack dopamine transporters) med can increase dopamine neurotransmission in this part of the brain

Risperidone (Risperdal)
Atypical Antipsychotic (most “typical” of atypicals)
Side effects: EPS (dose-dependent), TD, significant increases in PRL (check PRL levels)
–> most dopaminergic of Atypicals

IM formulation is available

Amphetamines
drugs that stimulate neural activity, causing speeded-up body functions and associated energy and mood changes

Dextroamphetamine
CNS stimulant
Mech: Inc catecholamines at the synaptic cleft, especially NE and dopamine.
Use: ADHD, narcolepsy, apetite control

Atomoxetine (Strattera)

  • BBW

Risk of suicidal ideation; monitor for suicidal thinking or behavior, worsening, or unusual behavior

  • Contraindications
    Glaucoma, pheochromocytoma, MAO I use within past 14 days
  • Warning
    Rare, but severe hepatotoxicity (most within 120 days of start of treatment)
    SE
    Headache, insomnia, somnolence, dry mouth, nausea, abdominal pain, decrease in appetite, nausea, etc.
    Priapism
    NOTE
    do not open capsule – irritant
    CYPD 2D6 substrate
    watch out for LIVER PROBLEMS

Clonidine Hydrochloride
Catapres, Catapres TTS, Kapvay (Antihypertensive)

Dexmethylphenidate Hydrochloride
Focalin; Focalin XR Tablet,
capsule
CNS Stimulant
ADHD

Guanfacine Hydrochloride
Tenex, Intuniv
Antihypertensive/Antipsychotic

Lisdexamfetamine
Vyvanse
(ADHD, binge eating disorder)

Methylphenidate
Concerta, Ritalin, Metadate, Methylin CNS Agent for ADHD Yes

methylphenidate hydrocholoride
Methylphenidate

Common brand names: Ritalin, Daytrana

This medication is used to treat attention deficit hyperactivity disorder – ADHD.

Brand names: Ritalin · Daytrana · Metadate ER · Methylin · Quillivant XR

Drug classes: Central Nervous System Stimulant · CNS Stimulant

Dissociative disorders
=A group of disorders that cause an impaired awareness of your own actions, thoughts, physical sensations, and even identity, which is a sense of who you are.

Dissociative disorders
Divided into three main types: depersonalization/derealization disorder, dissociative amnesia, and dissociative identity disorder.

Dissociative disorders
Each of these disorders fall along a spectrum of severity:
-depersonalization/derealization= the least severe of the dissociative disorders,
-dissociative amnesia= somewhere in the middle of least severe and severe,
-dissociative identity disorder= the most severe.

Depersonalization/derealization
-Might speak in a deadpan manner, with little emotion, and have trouble forming relationships.

-In severe cases, a person may have trouble recognizing familiar places, people, or objects, and this can make it had to learn tasks.

Caprylidene
Medical food approved by FDA for treatment of (AD)
Axona

Aducanumab

  • Alzheimer’s drug
  • decreases amyloid plaques

Donepezil (Aricept)
acetylcholinesterase inhibitor – inactivates enzyme that breaks down acetylcholine – which increases available acetylcholine – which slows progression of AD;
SIDE EFFECTS: think parasympathetic activation – bradycardia, hypotension, urination, defecation, GI upset, salivation, sweating

Galantamine
Reminyl ER
Cholinesterase Inhibitor
Dementia

Memantine (Namenda)
NMDA receptor antagonist

  • Alzheimer’s /disease

Rivastigmine

  • Acetylcholinesterase inhibitor
  • Used to tx Alzheimer’s
  • Increases amount of ACh in presynaptic terminals
  • Orally available
  • Very short T 1/2 (1.5 hrs)
  • Metab. by plasma cholinesterase **
  • Adverse effects: mostly safe (tremors, bradycardia, nausea, vomiting, etc. due to peripheral ACh stimulation)

Nuplazid
pimavanserin
Parkinsons Dx-hallucinations and delusions

Rivastigmine

  • Acetylcholinesterase inhibitor
  • Used to tx Alzheimer’s
  • Increases amount of ACh in presynaptic terminals
  • Orally available
  • Very short T 1/2 (1.5 hrs)
  • Metab. by plasma cholinesterase **
  • Adverse effects: mostly safe (tremors, bradycardia, nausea, vomiting, etc. due to peripheral ACh stimulation)

Pimavanserin
Nuplazid
Atypical antipsychotic used in Parkinson’s Disease
High affinity for serotonin 2A receptors

Question 3:
In which demographic is depression twice as prevalent in girls as compared to boys?
A. Preschoolers

B. School aged

C. Adolescents

D. All children

Which of the following is not a true statement with respect to confidentiality of the child or adolescent client?
A. The PMHNP should not be concerned with consent for disclosure when child abuse or maltreatment has occurred.

B. In 1979, the American Psychiatric Association (APA) stated that children 12 years of age or older can give consent for disclosure.

C. The American Academy of Child and Adolescent Psychiatry (AACAP) Code of Ethics states that consent is not required for disclosure.

D. Regardless of code or position statement by any organization, the best approach is when the child and PMHNP agree on disclosure.

Question 4:
Andrew is a 14-year-old male who is being managed for bipolar I disorder. He was started on lithium 6 weeks ago and has achieved a serum level of 1.1 mEq/L according to his most recent blood work. Andrew says he doesn’t feel any different, but both his parents and teachers report improvement in his mood. He has been more stable, is getting along better with friends and siblings, and is even more interested in his schoolwork. The PMHNP plans to maintain Andrew on this medication and knows that he will need which of the following ongoing laboratory assessments?
A. Complete blood count, thyroid function tests, and serum calcium

B. Liver function tests, complete blood count, and 12-lead electrocardiogram

C. White blood cell differential, fasting glucose, and fasting lipid profile

D. Comprehensive metabolic panel, complete blood count, and thyroid function tests

Question 5:
The therapeutic outcomes for children with disorders of written expression are most favorable when they are characterized by:
A. Concomitant pharmacotherapy with a psychostimulant to promote attention and focus

B. Multimodal therapy to include group interaction with peer-to-peer feedback on writing samples

C. variety of tutors who will offer a variety of writing techniques, composition strategies, and critiques

D. Intensive, continuous administration of individually tailored, one-on-one expressive and creative writing therapy

Aripiprazole
abilify/ antipsychotic (atypical)
S/sx: drowsiness

Risperidone
Atypical antipsychotic causing high prolactin levels
with lots of extrapyramidal SE

Amphetamine IR
Adderall
CNS Stimulant

Amphetamine XR
Adderall (XR) CNS Stimulant

Dextroamphetamine
CNS stimulant
Mech: Inc catecholamines at the synaptic cleft, especially NE and dopamine.
Use: ADHD, narcolepsy, apetite control

Atomoxetine (Strattera)

  • BBW

Risk of suicidal ideation; monitor for suicidal thinking or behavior, worsening, or unusual behavior

  • Contraindications
    Glaucoma, pheochromocytoma, MAO I use within past 14 days
  • Warning
    Rare, but severe hepatotoxicity (most within 120 days of start of treatment)
    SE
    Headache, insomnia, somnolence, dry mouth, nausea, abdominal pain, decrease in appetite, nausea, etc.
    Priapism
    NOTE
    do not open capsule – irritant
    CYPD 2D6 substrate
    watch out for LIVER PROBLEMS

Clonidine
alpha 2 agonist
-treatment for hypertension and opioid withdrawal
-side effects of bradycardia and hypotension

Dexmethylphenidate
ADHD stimulant

Guanfacine Hydrochloride
*Antihypertensive/Antipsychotic
*Tenex, Intuniv

Lisdexamfetamine
Vyvanse
ADHD
C-II

Methylphenidate
Concerta, Ritalin, Metadate, Methylin CNS Agent for ADHD

Methylphenidate ER
Concerta, Metadate CD, Ritalin LA, Quillichew (chewable tablets), Quillivant (suspension), Cotempla (ODT)

Question 6:
When considering a diagnosis of developmental coordination disorder, the PMHNP knows that the diagnosis may be associated with:
A. Above-average scores on performance subtests of standardized intelligence testing

B. Below-average scores on verbal subtests of standardized intelligence testing

C. Soft neurologic signs on physical examination such as slight reflex abnormalities

D. Physical findings consistent with neuromuscular disease such as muscular dystrophy

Question 8:
Children with gender dysphoria typically have higher rates of all the following except:
A. Depression

B. Impulse control disorders

C. Anxiety disorders

D. Eating disorder

Question 9:
Eileen is a 23-month-old girl who is being evaluated for autism spectrum disorder because her pediatrician is concerned about the presence of developmental red flags. She has just a few words of speech and has not put together any meaningful two-word phrases. While taking a history from Eileen’s mother the PMHNP learns that for approximately the last 2 months Eileen has been seen eating paint chips that are peeling off the baseboard and window sills in the family home; when she sees one she puts it in her mouth. This is a concern because they live in an old farmhouse and there may be lead-based paint in some of the paint layers. The PMHNP considers that which of the following is not consistent with a diagnosis of pica?
A. A diagnosis of autism spectrum disorder

B. Symptoms < 6 months duration

C. The pattern of eating

D. Lack of potential consequences

Question 12
Which of the following symptom clusters is most likely in a 16-year-old male with major depressive disorder?
A. Disturbance of mood, loss of interest, and mood-congruent hallucinations

B. Irritability, persistent abdominal complaints, and insomnia

C. Decreased concentration, social withdrawal, and substance abuse

D. Pervasive anhedonia, hopelessness, and severe psychomotor retardation

Question 10:
The current scholarly consensus is that the etiology of bulimia nervosa is multifactorial to include biological, sociocultural, family, cognitive-behavioral, and psychodynamic factors. When considering the etiology of bulimia nervosa, the PMHNP understands that:
A. Family factors contributing to bulimia nervosa include emotional restraint, tight parental control of behavior, and avoidance of conflict.

B. Lowered serotonin activity is seen in the brains of patients with bulimia nervosa.

C. There is a disproportionate incidence in childhood sexual abuse in patients with bulimia nervosa.

D. Cultural influences are much less implicit in the development of bulimia nervosa as compared to anorexia nervosa.

Question 11
Standardized instructional programs used in the management of reading disorders include all the following except:
A. The Orton Gillingham and Direct Instructional System for Teaching and Remediation

B. Science Research Associates, Inc. Basic Reading Program

C. Bridge Reading Program

D. Woodcock-Johnson Psychoeducational Battery

Question 13
The PMHNP is working with rural primary care providers to increase awareness of mental health disorders in infancy and early childhood. The program includes a session on screening for feeding disorders in infants. If an infant is either observed by the provider or reported by the parent to frequently suck the tongue rhythmically or appear to strain with his or her back arched and then swallow, the examiner should consider the possibility of:
A. Gastroesophageal reflux disease

B. Failure to thrive

C. Iron deficiency

D. Inadequate emotional attachment

Question 14
An astute pediatrician has referred Kyle, a 5-month-old infant, for evaluation of rumination disorder. His mother was concerned because he seemed to “want” to regurgitate every time he ate and would even seem to “put his hand down his throat” in order to stimulate regurgitation. Kyle had a thorough evaluation and according to the pediatrician did not have gastroesophageal reflux or hiatal hernia. The mother-infant relationship appeared to be healthy and, via assessment, the PMHNP could not identify any clear emotional issues. The primary treatment strategy for Kyle will likely focus on:
A. Aversive therapies, e.g., squirting lemon juice into Kyle’s mouth when he ruminates

B. Medication therapies to include trials of first-line agents such as metoclopramide, cimetidine, or haloperidol

C. Psychotherapy for the parents

D. Habit-reversal therapies

Question 15:
The literature indicates that boys whose fathers died before the age of 13 are at a greater risk for the development of depression as compared to controls. This is consistent with which general theory of depression etiology?
A. Molecular-genetic

B. Biologic

C. Psychosocial

D. Stress-diathesis

Question 16:
Katelyn is a 16-year-old girl who presents for therapy with her mother and father. Katelyn was born with male genitalia but has felt like a female “all of her life.” She says she knew something was different as far back as she can remember. She always wanted to wear her mother’s clothes and makeup and play with other girls. Katelyn started dressing and openly identifying as a girl when she was 13 years old, and her parents are trying to be supportive but they are struggling. Most recently Katelyn has developed an intimate partner relationship with Jennifer, a 15-year- old girl who was gender-assigned female at birth and identifies as a female. Katelyn’s father does not understand the relationship. The PMHNP explains that Katelyn:
A. Is responding to the male hormonal surges of puberty and will ultimately identify with her physiologic gender

B. Does not have a clear sense of gender identity and may be a candidate for reparative therapy to correct her gender identity issues

C. Is a transgender female who identifies as lesbian

D. Is a heterosexual male with transvestic disorder

Question 17:
All of the following are proposed etiologies of pica except:
A. Nutritional deficiencies

B. Parental neglect

C. Compensation for oral needs

D. Autism spectrum disorder

Question 19:
While the core features of schizophrenia are essentially the same in children as they are in adults, the presentation or characterization is sometimes very different given developmental issues. Unlike adults with schizophrenia, children with schizophrenia do not have:
A. Classic positive symptoms

B. Poverty of speech content

C. Social rejection

D. Delusions of persecution 11

Question 20:
The PMHNP is working with a couple who has been trying for years to conceive and is now ready to pursue adoption as an option. They are considering all possibilities; private vs. government-mediated adoption, adopting from another country, adopting a child of a different race or ethnicity, adopting an older child rather than an infant, and adopting a child who is currently in foster care vs. one who lives in an orphanage. While counseling this couple, the PMHNP advises them that:
A. Younger children adopted from foster care settings have the best outcomes

B. Children adopted from abuse and neglect situations have better psychological adaptation than those adopted as newborns

C. Transracial adoptions have demonstrated greater disruptions as compared to same-race adoptions

D. Adoptions into families as an only child are more successful than those into families with biological children

Question 21:
Mel is a 15-year-old male with a complex psychiatric assessment. He has a long history of mood instability, behavior problems, and trouble with schoolwork. He was an extremely active child who, at one time, was put on medication for hyperactivity. After a comprehensive evaluation, the PMHP recognizes that he meets diagnostic criteria for both bipolar I disorder and attention deficit hyperactivity disorder (ADHD). The most successful approach to his management is likely to begin with:
A. Pharmacotherapy for ADHD

B. Pharmacotherapy for bipolar I

C. Parental training and behavioral interventions for ADHD

D. Family-focused psychoeducational (FFT-A) for bipolar I

Question 22:
Bruce and Debbie have made an appointment for a psychiatric consultation for what amounts to psychiatric genetic counseling. Bruce is 31 years old and Debbie is 28 years old. They have been married for 3 years and want to have children. Debbie is worried because she has a diagnosis of schizophrenia. Her symptoms first became evident when she was in her junior year of college; fortunately, she knew about the disease due to family history and sought care quickly. She has been very open with Bruce, and they are both well informed about the disease. Debbie is an only child and her father had schizophrenia that was not well controlled. He committed suicide when she was 15 years old. Debbie takes olanzapine and feels well overall. She has a part-time job, functions well, but is concerned about the genetic nature of her disease. Counseling for Bruce and Debbie includes which of the following pieces of information?
A. Schizophrenia is up to eight times more prevalent in first-degree relatives as compared to the general population.

B. Schizophrenia has been isolated to chromosome number 5 and antenatal testing is available to predict genetic predisposition of the fetus.

C. Debbie’s age of onset is more highly correlated with incidence in her offspring as compared to early-onset schizophrenia.

D. Environment is at least as important as genetics with respect to expression of schizophrenic symptoms.

Question 23:
The PMHNP is working with a pediatrician colleague on a journal article to increase awareness and improve diagnostic strategies for early-onset bipolar disorder. Based upon a review of the longitudinal research on this disorder, the literature review of this article should include all the following except:
A. Recovery rates are lower when bipolar disorder has an early-childhood onset

B. Early-onset bipolar disorder presents a greater likelihood of mixed states and rapid cycling as compared to adolescent and adult onset

C. Higher socioeconomic status and lifetime psychosis are predictors of more rapid cycling in early-onset bipolar disorder

D. There is a more frequent conversion from bipolar II to bipolar I in childhood as compared to adults

Question 24:
Treatment of early-onset schizophrenia can be challenging due to the paucity of evidence-based support for various pharmacotherapeutics and nonpharmacologic interventions. In a recent study comparing olanzapine to clozapine, outcomes were assessed with the Clinical Global Impression of Severity of Symptoms Scale and Schedule for the Assessment of Negative/Positive Symptoms. Clozapine was found to demonstrate statistically significant superiority in which outcome measure?
A. Alleviating negative symptoms

B. Occurrence of extrapyramidal effects

C. Sustained remission of symptoms

D. Remission of hallucinations and delusions

Question 25:
Benjamin is a 4-year-old male who is being evaluated because his mom is afraid that he may be schizophrenic. There is a history of schizophrenia in the mother’s family; her maternal aunt, one sister, and one of her brother’s children were all diagnosed with schizophrenia. Today she reports that Benjamin has episodes during which he appears to be hallucinating. Sometimes he seems to see things that are not there, and other times he actually talks with and plays with imaginary people. The PMHNP continues to gather information from the mother and Benjamin and plans to do some observation of Benjamin during play. When considering a diagnosis of early-onset schizophrenia, the PMHNP considers all of the following except:
A.

B. Hallucinations are significantly more predictive of schizophrenia than delusions in young children

C. Benjamin’s symptoms are more likely a consequence of developmental immaturity

D. Benjamin’s family history should always be considered, but at this point the diagnosis is very unlikely

Question 26:
While counseling the parents of Joshua, a 9-year-old patient with developmental coordination disorder, the PMHNP advised that the treatment will include:
A. Modified physical education

B. Cognitive behavioral interventions

C. Group sports participation

D. Computer-generated developmental exercises

Question 27:
Maria is an 11-year-old girl who has been diagnosed with early-onset schizophrenia. In writing up her case report for publication, the PMHNP makes it a point to highlight which of the following historical features that are often seen in schizophrenia of middle childhood?
A. Onset of hallucinations and delusions before age 6

B. Symptoms easily confused with attention deficit hyperactivity disorder

C. Delayed motor milestones and language acquisition D. High expressed emotion characterized by overly critical family response

Question 28:
Ramon is a 12-year-old male who was started on fluoxetine, 40 mg daily, for major depressive disorder 6 weeks ago. At his 2-week follow-up, he was feeling well with no adverse drug effects, and his dose was elevated from 20 to 40 mg. Today he presents for a routine follow-up visit. He reports that he has been “all wound up” and is frequently agitated. He cannot sleep well and has trouble focusing at school. His friends say that he gets upset quickly, and they are starting to avoid him. The most appropriate approach is to:
A. Taper the fluoxetine off and see if these symptoms improve

B. Add a mood stabilizer to the fluoxetine

C. Increase his dose of fluoxetine to 60 mg daily

D. Continue the current dose and reassess in 2 weeks

Question 29:
Taryn is a 14-year-old female who is being evaluated as a condition to return to the private school in which she has been enrolled since fifth grade. She has an ongoing inability to focus in school, and her mood reportedly ranges from “one extreme to the other.” Per her mother, she is either overly excited about something and almost exaggerated in her approach, or she is disinterested and irritable and doesn’t want to leave her bedroom. She was evaluated several different times in childhood for ADHD, and even took medication once, but it did not help. In considering a diagnosis of cyclothymic disorder, the PMHNP considers that:
A. Symptoms need to be present for at least 1 year

B. Symptoms need to be present for at least 2 years

C. Symptoms are not characterized by conduct disorders

D. Symptoms are not characterized by suicidal ideation

Question 30:
Which of the following statements best characterized the treatment course and progression of bulimia nervosa?
A. Cognitive behavioral therapy is considered the benchmark, first-line treatment.

B. Antidepressant medications have not been demonstrated to be effective.

C. The majority of patients will require hospitalization as an initial intervention.

D. Psychodynamic therapy is extremely successful when patients are not lost to follow-up.

Question 31:
The etiology of childhood depression is multifactorial and may include biological factors. Which of the following is a true statement with respect to hormonal studies in depressed children?
A. Prepubertal children having a depressive episode secrete significantly more growth hormone during sleep than nondepressed children.

B. Polysomnography demonstrates significantly reduced rapid eye movement (REM) during depressive episodes. C. Thyroid stimulating hormone (TSH) levels are elevated > 50% above the upper limits of normal in the majority of children with depression.

D. Magnetic resonance imaging (MRI) studies in depressed adolescents reveals low ventricular volume and increased frontal lobe volume.

Question 32:
There is a high incidence of overlap among children with bipolar disorder, attention deficit hyperactivity disorder, conduct disorder, and anxiety disorders. Which of the following manic symptoms of bipolar disorder are most closely correlated to conduct disorder?
A. Insomnia and irritability

B. Physical restlessness and poor judgment

C. Hallucinations and overeating

D. Hyper sexuality and inattention

Question 33:
Linda is a 5-year-old girl who has persistent pica; she was finally referred for care when her eating of potentially toxic nonfood substances alarmed her kindergarten teacher. Linda’s mother admits during the history that Linda has been doing this for years, but thought it was not a big deal since Linda did not eat anything dangerous. Linda’s mother must work two Jobs and essentially did not bother to pursue Linda’s unusual symptom because it did not seem unsafe. While discussing management strategies with Linda’s mother, the PMHNP counsels that the most rapidly successful treatment strategy appears to be:
A. Aversion therapy

B. Play therapy

C. Environmental control

D. Cognitive behavioral therapy

Question 34:
The leading cause of death in youths living in juvenile residential facilities is:
A. Suicide

B. Accident

C. Homicide

D. Illness

Question 35:
Debbie is a 10-year-old female who has been referred to remediation therapy for her reading disorder. While designing her treatment program, the PMHNP knows that the most current strategies are characterized by:
A. An Individual Education Program (IEP) provided by the public school system

B. Continuous practice with flash cards, workbooks, and computer games

C. Focusing the child’s attention to the connections between speech sounds and spelling

D. Direct practice in spelling and sentence writing and review of grammatical rules

Question 36:
Justin is a 12-year-old male who was recently diagnosed with schizophrenia. He was quickly placed into a highly regarded assessment and treatment program and began pharmacotherapy and cognitive behavioral therapy. His parents have had a difficult time with the diagnosis as Justin has always been very healthy, a good school performer, and has never had any developmental concerns or delays. However, they are very supportive and committed to his recovery. Justin has been on an atypical antipsychotic for 1 month with no intolerable adverse effects. When counseling Justin’s parents about the prognosis, the PMHNP advises the parents that which of the following is more correlated with good outcomes in patients like Justin?
A. His age at the time of diagnosis

B. The absence of family history

C. Justin’s level of function before diagnosis

D. Tolerance of antipsychotic medication

Question 37:
Mrs. Henderson is a 24-year-old mother of 4 children under the age off 5. She has developed a trust relationship with the PMNHP after successful evaluation and management of ADHD in her oldest child. She now brings in her 3-year-old for an evaluation because she keeps eating things she finds within reach – paper, dirt, and one day, the mother found this child eating from the cat litter box. The mother says the child is up to date on her vaccines but she has not mentioned this problem to the pediatrician. The PMHNP knows that immediate assessment must include:
A. A comprehensive family assessment to include all children

B. Laboratory assessment of hemogram, iron, zinc, and lead levels

C. Wechsler Preschool and Primary Scale of Intelligence – Revised

D. Assessment of developmental milestones

Question 38:
Kelly is a 13-year-old female who is being evaluated at the recommendation of her seventh- grade teacher. This is her first year in junior high and her teacher is concerned over what appears to be marked social isolation. She does not appear to have any close friends or social contacts. She eats lunch with the girl who lives next door to her, but even that is initiated more by the other child. Her teacher also reported that Kelly seems to have certain unusual preoccupations, such a preoccupation with religions despite the fact that her family has never been religious. During the examination, Kelly clearly demonstrates some odd behavior. When talking about her family, she does not express any emotion. She does not laugh at anything and appears withdrawn, yet she will answer questions asked of her. The PMHNP, after completing his evaluation, considers a diagnosis of schizotypal disorder because review of Kelly’s assessment reveals:
A. The absence of any overt psychotic symptoms in the history

B. A family history of schizotypal disorder

C. An inability to be swayed from distorted perceptions D. Neglect of personal hygiene

Question 39:
The PMHNP is preparing a presentation for a conference of pediatric primary care providers. The topic of the presentation is early identification and referral of developmental coordination disorder. When outlining high-risk populations, the PMHNP discusses that statistically there is a higher incidence of occurrence in children with all of the following except:
A. Speech disorders

B. Learning disorders

C. Attention deficit hyperactivity disorder (ADHD)

D. Oppositional defiant disorder (ODD)

Question 40:
Margaret is a 14-year-old girl being seen in follow-up for major depressive disorder. She has been on a therapeutic dose of a selective serotonin reuptake inhibitor (SSRI) for 3 months and has still failed to achieve remission. Consistent with the Texas Children’s Medication Algorithm Project (TMAP), the next action should be to:
A. Change to an SNRI

B. Change to an atypical antidepressant

C. Change to bupropion

D. Change to another SSRI

Question 41
The PMHNP is developing a brief informational pamphlet on gender dysphoria to distribute at a local mental health awareness event for the public. The “basic facts” section of the pamphlet correctly states that:
A. The majority of children who demonstrate nonconforming gender behavior in childhood grow up to be transgender adults

B. The ratio of gender dysphoria in adolescent boys and girls is equal

C. A genetic basis for gender dysphoria has been identified

D. Diagnostic criteria for gender dysphoria is the same across the lifespan

Question 42
The American Adoption Congress represents the shared interests of the adoption triad, a phrase used to represent:
A. Adoptive parents, siblings, and adoptees

B. Birth parents, adoptive parents, and adoptees

C. Adoptive parents, birth parents, and state government

D. Birth parents, adoptees, and federal government

Question 43:
Susan is a 12-year-old girl who is referred for psychiatric evaluation because she is having social problems at school. She is always picked last for teams in physical education, and she is becoming exceedingly depressed about the lack of social interaction at school. A detailed history reveals that Susan has always had delayed developmental milestones, including delayed sitting without support and transferring objects hand to hand. She did not walk until she was 20 months old, and for years afterward had persistent issues with falling. Now, at age 12, she is having trouble with her handwriting. The PMHNP considers which of the following diagnoses as most likely?
A. Disorder of written expression

B. Autism spectrum disorder

C. Developmental coordination disorder

D. Learning disorder not otherwise specified

Question 44:
Bipolar I disorder is being diagnosed with increasing frequency in prepubertal children. Which of the following is a true statement with respect to this trend?
A. Better awareness and screening practices promote earlier diagnosis and management.

B. The diagnosis is controversial because few children this age exhibit discrete mood cycles.

C. The majority of children diagnosed with bipolar I have a history of attention deficit hyperactivity disorder. D. Prepubertal children with bipolar I are extremely sensitive to mood stabilizers

Question 45:
Early-onset schizophrenia is quite rare as compared to adolescent and adult onset. Certain features are unique to early-onset schizophrenia and include all the following except:
A. Extremely high rates of comorbid psychiatric disorders

B. Marked deficits in intelligence quotient (IQ)

C. Significant delay in perceptuomotor skills

D. Better long-term outcomes with early intervention

Question 46:
Conventional antipsychotics are not first-line interventions in early-onset schizophrenia due to the risk of dystonic reactions. However, when children are not responsive to first-line therapy with atypical antipsychotics, which of the following is the most appropriate conventional choice?
A. Haloperidol

B. Trifluoperazine

C. Risperidone

D. Chlorpromazine

Question 47:
Cameron is a 7-year-old boy who has been diagnosed with ADHD and started on psychostimulants. Two months later there has been no meaningful improvement of symptoms, and he is referred from primary care to a PMHNP. While reviewing his records, the PMHNP notes that Cameron has been living with his aunt and uncle and their four children since the death of his mother 6 months ago. Cameron, who reportedly was always a happy and well- adjusted only child, developed symptoms shortly after these major life changes. His symptoms were characterized primarily by temper tantrums and an inability to sit still. He is not functioning well in school and is having repeated stomachaches and headaches. The PMHNP considers that the most appropriate action would be to stop the psychostimulant and assess Cameron for:
A. Bipolar I disorder

B. Agitated depression

C. Dysthymic disorder

D. Cyclothymic disorder

Question 48:
When counseling the parents of an intersex neonate, the PMHNP recognizes that the current standard of care is to counsel toward:
A. Male-to-female surgical procedures

B. Flexible sex rearing

C. An early and consistent decision of the child’s gender D. Hormone augmentation of chromosomal gender

Question 49:
In the juvenile justice system, the difference between delinquent acts and status offenses is:
A. Delinquent acts require detention; status offenses may be remediated in other ways

B. Delinquent acts are ordinary crimes committed by juveniles; status offenses would not be crimes if committed by an adult

C. Delinquent acts are the first offense; status offenses are repeat commissions of the same act

D. Delinquent acts require jury trial; status offenses are adjudicated by a judge

Question 50:
There are several physiologic abnormalities that may result in ambiguous genitalia and/or an unclear sense of gender identity leading to gender dysphoria. When an adolescent female is found to have cryptorchid testes, this indicates a condition known as:
A. Congenital adrenal hyperplasia

B. Androgen insensitivity syndrome

C. Turner’s syndrome

D. Klinefelter’s syndrome

Question 51:
Which of the following is a true statement with respect to developmental coordination disorder?
A. The majority of children with this disorder do not have a history of significant developmental milestone delay.

B. The clumsiness of developmental disorder generally persists into adolescence and adulthood.

C. Most successful treatment strategies involve integrative physical education to include the peer group in team sports.

D. Neurologic examination is very similar to patients with neuromuscular disease.

Question 52:
Donna is a 16-year-old transgender female who has been through extensive individual and family counseling and is ready to start hormone therapy with estrogen, progesterone, and testosterone-blocking agents. When counseling her specifically about the risks, benefits, and required monitoring of hormonal therapy, the PMHNP advises Donna that:
A. Her voice will become more consistent with the female gender

B. She will need routine lipid and diabetes screening

C. Sterility is a probable consequence of hormone therapy

D. Worsening of acne is common in the first year of treatment

Question 53:
The difference between hormonal management of transgender adolescents from adults is that which of the following has no role in adult management?
A. Estrogen

B. Gonadotropin-releasing hormone

C. Testosterone

D. Testosterone blockers

Question 54:
Jessica is a 26-month-old female who is being evaluated because she will not eat. Her parents report that she just flat out will not eat her meals. Her mother says that she has followed all the pediatrician’s suggestions: she has eliminated any snacks or drinks between meals, and she has offered a variety of foods, including those that Jessica seemed to enjoy previously. Jessica is not sick; has no problems with vomiting or elimination abnormalities. Jessica’s parent say that her pediatrician is not concerned, but they are not comfortable with what appears to be an almost complete absence of food intake. While considering a diagnosis of avoidant food intake disorder the PMHNP knows that any of the following would fulfill the diagnostic criteria except:
A. An irritable, apathetic, or anxious affect

B. Failure to gain weight consistent with her trajectory since birth

C. A loss of 15% body weight

D. Use of daily nutritional supplements

Question 55:
Rose is a 13-year-old girl who is being evaluated as part of a family assessment; the primary patient is Rose’s 8-year-old brother who is demonstrating behavior of concern and is having a Court-ordered evaluation. During the family assessment, it becomes apparent that Rose’s mother is very concerned that Rose is a tomboy. The mother, who is very elegant, is distressed by Rose’s persistent “tomboy” behavior and worries that Rose might become a lesbian, which would be “unacceptable” to the family. More detailed evaluation of Rose reveals that she is experiencing some sexual reflection. She excels at sports and has always preferred rough and tumble play, but she doesn’t see anything wrong with that. She thinks she is sexually attracted to one of her female teachers, and sometimes fantasizes about her. Rose just began menstruating 3 months ago, and while she has had a boyfriend at school, she is not sexually active in any way; they have kissed a few times, and she likes it, but she has no plans to take it any further. Otherwise, Rose seems well adjusted, worries about her brother, and dismisses her mother’s concerns as “silly.” Which of the following statements best characterizes Rose?
A. Rose should be evaluated for gender dysphoria as she meets the criteria of “strong preference for activities stereotypically engaged in by the other gender.”

B. Rose is probably a lesbian and family therapy should include working with the mother to accept Rose’s sexual orientation.

C. Rose may be a candidate for conversion therapy as her preferences are ambiguous at this point in her development.

D. Rose does not appear to have concerns about her gender identity of sexuality and no further evaluation is indicated.

Question 56:
Learning disorders affect at least 5% of all school-aged children in the United States. Since 1975, Public Law 94-142 mandates that all states provide free, appropriate services to all children. Among the various types of learning disorders, the PMHNP knows that the overwhelming majority are:
A. Reading disorders

B. Mathematics disorders

C. Disorders of written expression

D. Learning disorders not otherwise specified (NOS)

Question 57:
The PMHNP has been trained in custody evaluations and is preparing to perform his first evaluation as a guardian ad litem. The case involves a 6-year-old boy and his 4-year-old sister. The parents are very angry and not able to talk or come to any agreements at all. Both parents want full custody and support from the other parent, both of whom are working professionals. After interviewing each party alone and then conducting a family interview, the PMHNP reviews all records made available, including the legal filings and petitions. There are no allegations of abuse or neglect or unsuitability from either parent; they just each want full custody. In addition to considering the best interests of the children, the PMHNP knows that the elements considered by the court will include all the following except the:
A. Children’s current adjustment to home, school, and community

B. Wishes of the children and parents

C. Physical health of parents and children

D. Parent’s degree of financial resources

Question 58:
Marion is a 17-year-old female who has been referred by her high school guidance counselor for evaluation. The counselor is concerned that Marion has an eating disorder because she has seen her in the bathroom on several occasions vomiting, but there is no other indicator of illness like fever or missing school days. When considering the diagnosis of anorexia nervosa, the PMHNP knows that all of the following must be present except:
A. A voluntary, unhealthy degree of weight loss and maintenance

B. An intense fear of becoming fat

C. Some form of disorder of menstruation

D. Symptoms present for at least 3 months

Question 59:
Ryan is a 6-year-old male who is being evaluated because his pediatrician is concerned that he demonstrates a marked inability to perform the daily motor skills consistent with what is expected at his age. At the age of 3 he was assessed due to delay in developmental milestones and was found to have an IQ of 68, consistent with mild mental retardation. When interpreting his motor coordination today, the PMHNP considers that:
A. Deficits in coordination are consistent with mental retardation; mental retardation precludes a diagnosis of developmental coordination disorder

B. This diagnosis is unlikely for Ryan as developmental coordination disorder is almost exclusively a diagnosis of females

C. Gross motor problems are often associate with comorbid language disturbance

D. Secondary peer relationship problems are common in children with developmental coordination disorder

Question 60:
Kelly is a 14-year-old female who has finally been referred for management of anorexia nervosa. She was diagnosed almost 1 year ago with the food-restricting subtype, but attempts to get her into psychiatric care were unsuccessful. She continues to be resistant but her caloric intake is now < 400 daily and she finally appears to be unable to sustain the supraphysiologic levels of exercise that she has maintained to try and “keep her weight down.” She is 5’2″ tall and weighs 82 lbs., which is approximately 75% of ideal body weight for her height. Her vital signs are stable and surprisingly there are no profound laboratory or ECG abnormalities. When counseling Kelly and her parents about the recommended course of treatment, the PMHNP advises that Kelly will require:
A. Intensive dynamic psychotherapy to alter eating behavior

B. Hospitalization for controlled weight gain

C. Pharmacotherapy with selective serotonin reuptake inhibitors

D. Family therapy to include all members living in the home

Question 61:
Rumination is a feeding disorder most commonly seen in infants, but it can occur at any point in the lifespan. Characteristic findings in infants include:
A. Resultant soothing

B. Comorbid hiatal hernia

C. Failure to thrive

D. Onset at 4-6 weeks of age

neural tube defects
malformations of the brain, spinal cord, or both during embryonic development that often result in lifelong disability or death

Question 62:
Ms. Stevenson is a 21-year-old woman who brings her 3-week-old infant to the PMHNP for an “emergency” evaluation. Ms. Stevenson has a 2-year-old who was diagnosed with rumination disorder when he was 10 months old and had to be hospitalized for tube feedings while the family started treatment. Ms. Stevenson is worried now because her new baby vomits every time he eats; she is afraid he has the same thing. The PMHNP counsels Ms. Stevenson that:
A. Rumination does not run in families and it is unlikely that the new baby has this rare disorder.

B. The baby needs to be evaluated for pyloric stenosis and should be referred to his pediatrician.

C. The family will likely benefit from the same psychotherapeutic interventions that were used with the older child.

D. They will begin with a nutritional assessment to see if the new baby needs a feeding tube.

D. They will begin with a nutritional assessment to see if the new baby needs a feeding tube.

Question 63:
Which of the following is a true statement with respect to avoidant/restrictive food intake disorder?
A. Onset at less than 1 year of age is correlated with growth delay.

B. Approximately 70% of infants who persistently refuse food continue to have some eating problems during childhood.

C. In older children and adolescents, this disorder typically resolves spontaneously.

D. The diagnosis may be appropriate even without evidence of nutritional deficiency.

Question 64:
Roshan is a fourth grader who is being evaluated for poor scholastic performance in mathematics. He has always been a happy, healthy child, has socialized with friends at school, and presents no behavioral concerns at home. However, his math performance has always been below the average for his grade, and now he is performing so far below his peers that he is really beginning to be upset about it. While being evaluated for a learning disorder, the PMHNP appreciates that Roshan has had a marked deficit in his ability to recognize and understand symbols and order clusters of numbers. This suggests a deficit in:
A. Linguistic skills

B. Perceptual skills

C. Mathematic skills

D. Attention skills

Question 65:
Regarding pediatric suicide, which of the following is a true statement?
A. Worldwide, suicide very rarely occurs in children who have not reached puberty.

B. In the last 15 years, both suicidal ideation and completed suicide rates have increased among adolescents.

C. Cognitive immaturity is significantly correlated with risk of completed suicide in children of all ages.

D. Approximately 75% of suicidal children communicate intent or ideation to a friend or relative within 24 hours of the attempt.

Question 66:
The PMHNP is having a family meeting with 8-year-old Hunter and his father and stepmother to discuss the results of his mathematics assessment. Analysis of his performance on the KeyMath Diagnostic Arithmetic Test reveals a performance markedly below what is expected for his age. Hunter’s mother feels certain that his poor performance is a result of poor education; apparently Hunter’s mother was homeschooling him, but subsequently she was found to be so neglectful of all of his needs that his father and stepmother were given full custody. When discussing the course and prognosis for Hunter’s mathematics disorder, the PMHNP advises that:
A. Compared to other learning disorders, mathematics disorder does not appear to depend on the amount or quality of instruction

B. Mathematics disorders are not stable over time, and early intervention may lead to improvement of skills

C. The remediation program will emphasize computational skills

D. Pharmacotherapy with remediation produces the best outcomes

Question 67:
While not currently indicated for the treatment of early-onset bipolar disorder, which of the following medications has demonstrated utility in clinical trials without any associated weight change, rash, or other adverse events?
A. Lamotrigine

B. Olanzapine

C. Valproic acid

D. Lithium

Question 68:
Rose is a 12-year-old female who is being evaluated for declining school performance and an increasing tendency to try to avoid going to school. Historically she has been an average student, although she has particularly struggled with writing exercises and has actually failed assignments this year. She consistently ignores rules of grammar, and her teacher says that her writing submissions look as if they were written by a much younger child. The PMHNP knows that the initial evaluation of Rose must include all the following except:
A. The Wechsler Intelligence Scale for Children III

B. Assessment for pervasive developmental disorder

C. The Test of Early Written Language

D. Screening for ADHD

Question 69:
All the following are true with respect to making a diagnosis of major depressive disorder in children except:
A. There must be a change from previous level of functioning.

B. There must be social or academic impairment.

C. There must be somatic or psychomotor complaints. D. There must be depressed or irritable mood.

Question 70:
Tiana is a 10-year-old girl who is being referred for evaluation because her school performance is appreciably below what is expected at her age. Historically she has been a very happy child, likes school, and looks forward to going. However, over the last several months her teacher reports that she is much slower than her peers in reading, and she appears to be upset and withdrawn when asked to read in class. The PMHNP would expect additional report from the teacher to include all the following except:
A. Inability to copy correctly from a printed text

B. Poorly established left-to-right tracking

C. Omissions and distortions of words when reading aloud

D. Compensation with use of memory and inference

Question 71:
The PMHNP has completed the initial assessment of Bruce, a 14-year-old boy who was initially presented to care by his parents because of progressive social withdrawal. Upon completion of the patient interview, mental status exam, and family assessment, review of all information reveals that for the past 15 months Bruce has been progressively “moody.” His parents say that he has become so irritable that his little brother and sister are afraid to talk to him anymore. Additionally, he has become socially withdrawn, now not even wanting to go to school. Bruce says he is tired all the time and just doesn’t feel like doing anything. He often does not complete homework assignments. When considering a diagnosis of dysthymic disorder, the PMHNP knows that which other history finding must be present?
A. Depressed mood most of the time for at least 2 weeks

B. Symptoms not resolved for more than 2 months at a time

C. Intermittent episodes of anger disproportionate to the stimulus

D. Concomitant substance abuse

Question 72:
The PMHNP is working with the parents of a 13-month-old male who is diagnosed with avoidant/restrictive food intake disorder. The toddler will eat, but he seems to tire both physically and emotionally very quickly and is easily distracted; once distracted, he will not return to the meal. His weight trajectory has dropped and he has gone from a weight of 60 percentile for age/height to 10 percentile for age/height. As part of the transactional plan of care, the parents will be encouraged to:
A. Allow the child to determine the eating schedule, and feed him whenever he appears hungry

B. Offer high calorie liquid supplements between scheduled meals

C. Present playful stimuli during and immediately after meals

D. Give attention and praise to positive eating behaviors

Question 73:
Jared is a 6-year-old boy who comes to the PMHNP for an evaluation with his father. The father reports that he is worried about Jared because he has had problems fitting in at school ever since he started kindergarten. He does not have any friends at school and does not seem to know how to play with others. Dad reports that Jared has never been “very talkative” and sometimes switches from one topic to another without any reason. When considering early-onset schizophrenia, the PMHNP recognizes that which of the following must be present?
A. Visual or auditory hallucinations

B. Deteriorating function over the last several months

C. A history of normal developmental milestones

D. Negative symptom onset before age 5 years

Question 74:
The PMHNP suspects that Wesley, an 8-year-old male, has a reading disorder. In kindergarten his teacher documented some suspicion for a disorder, but throughout first grade and now into second grade, he is clearly functioning below expected levels.. He becomes increasingly anxious when asked to read in school. Which aspect of Wesley’s history would support the risk for this diagnosis?
A. Visual perceptual deficits

B. Social anxiety disorder

C. Inadequate schooling

D. Attention deficit hyperactivity disorder

Question 75:
The PMHNP is treating Pam, a 13-year-old female, for moderate-to-severe major depressive disorder. In addition to cognitive behavioral therapy, the PMHNP discusses with the patient and her father the plan to begin sertraline, 50 mg daily, then titrate the dose up when tolerance is established. Pam’s father has researched this medication and is concerned because he read about the risk of increased suicidal ideation. The most appropriate response is to tell Pam’s father that:
A. His research is correct and they can use another drug class if he is more comfortable with that

B. The risk is actually decreased when sertraline is used with cognitive behavioral therapy

C. More recent research suggests that this is not accurate and that treatment actually decreases risk of suicide

D. Sertraline is the only drug in its class indicated for adolescents and it does not carry this risk

Akathisia

  • motor restlessness, need to keep going, tx with antiparkinsons meds, can be mistaken
    for agitation

Akinesia
loss or impairment of the power of voluntary movement

pseudo-parkinsonism
side effect of antipsychotic drug, rigidity, pill rolling, masked face, shuffling gait

Cogentin (benztropine)
Anticholinergic
Used in akinesia, akathisia, dystonia, pseudo-Parkinson’s

Benadryl (diphenhydramine)
Antihistamine used in akinesia, dystonia, pseudo-Parkinson’s

Symmetrel (amantadine)
Antiviral; also used for Parkinson’s Disease
Dopamine agonist used in akinesia and pseudo-Parkinson’s

Dantrium (dantrolene)
Peripheral acting skeletal muscle relaxant;
Interferes with calcium releases from skeletal muscle cells, used to reduce muscle rigidity (commonly used off-label for neuroleptic malignant syndrome, not Food and Drug Administration approved

Risk factors for neuroleptic malignant syndrome
-Rapid dose escalation
-Parental route of administration
-Higher potency (higher affinity for D2 blockade)
-Typical antipsychotics
*Emergent and focusers on cardiopulmonary support

Pseudo-Parkinson’s
-Shuffling gait, motor slowing, masked flat facial expression, tremor, and cogwheel rigidity

Extrapyramidal side effects (EPSE)
Monitor Abnormal Involuntary Movement Scale (AIMS)

1Question 1 The Confusion Assessment Methods Instrument (CAMI) is a standardized assessment tool for which of the following disorders? A. Dementia B. Delirium C. Pick’s disease D. A and CThe answer is B-Delirium Question 2 Which of the following is a common sensory deficit of conversion disorder? Check all that apply. A. Blindness B. Tunnel vision C. Blindness D. DeafnessThe answer is A-Blindness and D-Deafness Question 3 Delirium is an example of which type of clinical problem in psychosomatic medicine? A. Psychiatric symptoms secondary to a medical condition B. Psychiatric symptoms as a reaction to a medical condition of treatment C. Psychiatric complications of medical conditions and treatment D. Medical complications of psychiatric conditions or treatment The answer is A- Psychiatric symptoms secondary to a medical condition Question 4 All the following medications are used in the treatment of Parkinson’s disease dementia/psychosis EXCEPT for A . NuLaid B . Rivastigmine C. Pimavanserin D. Memantine The answer is D-Memantine Question 5Indicators of imitative dissociative identity disorder include which of the following symptoms? A. Symptom exaggeration B. Confusion and shame related to symptoms C. Use of symptoms to excuse antisocial behaviors D. A and C onlyThe answer is B- confusion and shame related to symptoms Question 6 Which of the following can lower a person’s resistance to control impulses? A. Fatigue NRNP 6665 Final Exam-with 100% verified solutions-2023-2024
2B. Incessant stimulation C. Psychic trauma D. All of the above The answer is D-All of the above Question 7 Schizophrenia in a patient with end-stage renal disease is an example of which type of clinical problem in psychosomatic medicine? A. Co-occurring medical and psychiatric conditions B. Psychiatric symptoms secondary to a medical condition C. Psychiatric symptoms as a reaction to a medical condition D. Psychiatric complications of medical conditions and treatments the answer is A-Co-occurring medical and psychiatric conditions Question 8 An acute onset, short-term confusion, with changes in cognition and level of awareness due to a physiological cause is known as which of the following? A. Delirium B. Dementia C. Psychosis D. Traumatic brain injury The answer is A -Delirium Question 9 A patient expresses feelings of unreality or of being detached from their environment, describing the perception of the outside world as unreal, dreamlike, and visually distorted. The ARNP recognizes this as which of the following? A. Derealization B. Depersonalization C. Generalized amnesia D. Dissociative identity disorder The answer is A-derealization Question 10 A patient present with persistent feelings of detachment from one’s self, like watching one’s self in a movie. The ARNP recognizes this as which of the following? A. Derealization B. Depersonalization C. Generalized amnesia D Dissociative identity disorderThe answer is B-Depersonalization Question 11 Which of the following is recommended in the treatment of pyromania? A. Psychoanalysis B. Cognitive therapy
3C. Supervision of patient to prevent a repeated episode of fire setting D. All the above The answer is B- Cognitive therapy Question 12 Which of the following biological factors have been associated with kleptomania? Check all that apply. A. Brain diseases B. Cortical atrophy C. Mental retardation D. Enlarged lateral ventricles The answers are: A-brain diseases, B-Cortical atrophy, and D-enlarged lateral ventricles Question 13 A neurodevelopmental disorder characterized by impairment confined to a specific area of academic achievement (i.e., reading, writing, arithmetic, spelling) without deficits in intellectual and adaptive behaviors is A. Intellectual disability B. Communication disorder C. Specific learning disorder D. Autism spectrum disorder The answer is A-Intellectual disabilityQuestion 14 Which of the following approaches/treatments are recommended in working with patients with a conversion disorder? A. After a very thorough evaluation to r/o any medical cause, tell the patient that the symptoms are imaginary. B. Recommend psychotherapy to focus on issues of stress and coping. C. Recommend psychoanalysis to explore intrapsychic conflicts. D. B and C only The answer is B -Recommend psychotherapy to focus on issues of stress and coping Question 15 Experiences of depersonalization and decreolization are common in which of the following patients? A. Patients with seizures B. Patients with migraines C. Patients who use marijuana D. All of the above The answer is C- Patients who use marijuana Question 16 Under hypnosis or during psychotherapy, a patient may recover a memory of a painful experience that is etiologically significant. This is known as which of the following? A. False memory syndrome B. Recalled memory syndrome
4 C. Dissociative trance disorder D. Recovered identity disorder The answer is C- Dissociative trance disorder Question 17 Which of the following is recommended as a first line therapy for a patient with dementia and behavioral disturbance? A Valproic acid (Depakote) B. Risperdal C. Haloperidol D. None of the above The answer is D-none of the above Question 18 According to the DSM-5, delirium is specified as acute or persistent. Which of the following time frames is consistent with acute delirium specification? A. A few hours or days B. Three weeks or less C. One month or less D. Less than 3 months The answer is A- A few hours or days Question 19 Which of the following is true about impulses? Check all that apply. A. Impulses are acted upon with the expectation of receiving pleasure B. Impulses are usually ego-dystonic. C. Impulsive behaviors are characterized by their repetitive nature. D. The repeated acting out of impulses leads to psychological impairment. The answers are B-, Impulses are usually ego-dystonic, C- Impulsive behaviors are characterized by their repetitive nature, and D-. The repeated acting out of impulses leads to psychological impairment Question 20 MRI findings in patients with intermittent explosive disorder may reveal changes to which area of the brain that is associated with loss of impulse control? A. Cerebellum B. Prefrontal cortex C Temporal lobeD. Parietal lobeThe answer is B-Prefrontal cortex Question 21 Patients with kleptomania have a high lifetime comorbidity of which of the following disorders? A. Mood disorders B. Schizophrenia C. Dissociative disorders
5D. All of the above The answer is A-Mood disorders Question 22 The epidemiology related to kleptomania includes which of the following? A. Kleptomania is more prevalent in males than females. B. Kleptomania is estimated to be about 10 percent in the general population. C. Kleptomania is reported to occur in fewer than 5 percent of identified shoplifters. D. All of the above The answer is A- Kleptomania is more prevalent in males than females Question 23 An increase in depersonalization is seen with the depletion of which of the following? A. GABA B. Serotonin C. L-tryptophan D. Norepinephrine The answer is B- Serotonin Question 24 Depression secondary to interferon treatments represents which of the following clinical problems in psychosomatic medicine? A. Psychiatric complications of medical conditions and treatments. B. Psychiatric symptoms secondary to a medical condition. C. Psychological factors precipitating medical symptoms. D. Psychiatric symptoms as a reaction to medical conditions or treatments.The answer is D- Psychiatric symptoms as a reaction to medical condition or treatments Question 25 The ARNP is meeting with a person who reports a fascination with fire, along with recurrent deliberate and purposeful setting of fires. The ARNP realizes that these behaviors are consistent with which of the following disorders? A. Pyromania B. Obsessive-compulsive disorder C. Intermittent explosive disorder D. Pyro phobia The answer is B- Obsessive-compulsive disorder Question 26 Which of the following is not consistent with what is known about depersonalization and derealization? A. They are the third most commonly reported psychiatric symptom. B. Derealization is two to four times more often in men than in women. C. One survey found a one-year prevalence of 19 percent in the general population. D. Transient experiences of depersonalization and derealization are extremely common in normal and clinical populations.
6The answer is A- They are the third most commonly reported psychiatric symptom Question 27 Which of the following are included in the clinical features of anxiety illness disorder? Check all that apply. A. Persons maintain they have a particular disease or as time progresses their belief may transfer to another disease. B. Lab results, lack of progression of the disease, and appropriate reassurances from the provider are helpful treatments for the person with the disease. C. Preoccupation with illness may or may not interfere with their interaction with family, friends, and co-workers. D. They are often addicted to internet search about their feared illness, inferring the worst from the information. The answers area- Persons maintain they have a particular disease or as time progresses their belief may transfer to another disease and D- They are often addicted to internet search about their feared illness, inferring the worst from the information Question 28 The ARNP in working with a parent of a 6-month-old would offer which of the following in anticipatory guidance? A. Give baby much attention.B. Attune to baby’s needs for hunger, fatigue, diaper change. C. Provide supervised time for crawling, sitting, and rolling. D. All the above The answer is C- Provide supervised time for crawling, sitting, and rolling Question 29 The symptom of giving approximate answers is known as which of the following? A. Alogia B. Paralogia C. Analogia D. Symlogia The answer is A-Logia Question 30 Identifying developmental milestones is an important skill set for ARNPs for which of the following reasons? A. Often developmental milestones go unrecognized until a child is much older. B. Knowing when a significant variation in development has occurred improves diagnostic accuracy.C The sooner a significant developmental impairment is identified and addressed, the. better the long-term outcomes could be for patients. D. All of the aboveThe answer is C- The sooner a significant developmental impairment is identified andaddressed, the. better the long-term outcomes could be for patients
Recommended for you

Document continues below
20
ATI med surg assessment-with 100% verified solutions-2023-2024
ATI med surg assessment-with 100% verified solutions-2023-2024
swift river
100% (9)
5
NRNP 6665 Midterm Exam 2023, Walden University, NRNP6665 Midterm Exam, NRNP 6665 Week 6 Midterm Exam, NRNP6665 Week 6 Midterm Exam
NRNP 6665 Midterm Exam 2023, Walden University, NRNP6665 Midterm Exam, NRNP 6665 Week 6 Midterm Exam, NRNP6665 Week 6 Midterm Exam
Medical
50% (6)
4
NRNP 6645 Final Exam 2023, Walden University, NRNP6645 Final Exam, NRNP 6645 Week 11 Final Exam, NRNP6645 Week 11 Final Exam
NRNP 6645 Final Exam 2023, Walden University, NRNP6645 Final Exam, NRNP 6645 Week 11 Final Exam, NRNP6645 Week 11 Final Exam
Medical
None
6
NRNP 6560 Final Exam 2023 (3 Versions), Walden University, NRNP6560 Final Exam, NRNP 6560 Week 11 Final Exam, NRNP6560 Week 11 Final Exam
NRNP 6560 Final Exam 2023 (3 Versions), Walden University, NRNP6560 Final Exam, NRNP 6560 Week 11 Final Exam, NRNP6560 Week 11 Final Exam
Medical
None
7 Question 31 Which of the following can cause delirium? Check all that apply. A. Polypharmacy B. Sleep deprivation C. Admission/transfer/discharge from a healthcare facility D. None of the above The answers are A- Polypharmacy, and C- Admission/transfer/discharge from a healthcare facility Question 32 A patient who has been raped, presents with the inability to recall important personal information and any information about the rape, does recall events prior and since. There does not appear to be any physiological reason for this. This presentation is consistent with which of the following diagnoses? A. Dissociative amnesia B. Dissociative amnesia with dissociative fugue C. Localized dissociative amnesia D. Generalized dissociative amnesiaThe answer is C-Localized dissociative amnesia Question 33 A term use to describe a neurodevelopmental disorder characterized by impairments in reciprocal social communication and a tendency to engage in repetitive stereotyped patterns of behaviors, interests and activities is A. Intellectual disability B. Specific learning disorder C. Autism spectrum disorder (ASD) D. Attention deficit/hyperactivity disorder (ADHD) The answer is C- Autism spectrum disorder (ASD) Question 34 Acute withdrawal from alcohol represents which type of clinical problem in psychosomatic medicine? A. Medical complications of psychiatric conditions or treatments B. Psychiatric complications of medical conditions and treatments C. Psychiatric symptoms as a reaction to medical treatments. D. Co-occurring medical and psychiatric conditions. The answer is B- Psychiatric complications of medical conditions and treatments Question 35 A dissociative disorder described as an identity disturbance due to prolonged and intense coercive persuasion is known as which of the following?A. Brainwashing B. Ganser syndrome C. Imitative dissociative identity disorder D. Factitious dissociative identity disorder
8The answer is A-Brainwashing question 36 Discrete episodes of losing control of aggressive impulses grossly out of proportion to any stressors, which can result in serious assault tor destruction of property are symptoms consistent with which of the following disorders? A. Conduct disorder B. Antisocial personality disorder C. Borderline personality disorder D. Intermittent explosive disorderThe answer is D- Intermittent explosive disorder Question 37 Epidemiology data suggests which of the following with respect to illness anxiety disorder? A. Illness anxiety disorder is more common in men than in women. B. Illness anxiety disorder is seen more frequently in persons with higher education. C. Surveys indicate that up to 15 percent of the general population worry about becoming sick and incapacitated as a result. D. Illness anxiety disorder is more common among caucasians than among other races. The answer is C- Surveys indicate that up to 15 percent of the general population worry about becoming sick and incapacitated as a result, Question 38 Which of the following treatments is NOT recommended in treating Dissociative Identity Disorder? A. Cognitive TherapyB. Hypnosis C. Antidepressants D. Group Psychotherapy The answer is D- Group Psychotherapy Question 39 A person reveals to the ARNP that they steal to get the things they want and cannot afford. The person says they just can’t help themselves, because they have kleptomania. The ARNP realizes the following: A. This presentation is consistent with kleptomania. B. This presentation is not consistent with kleptomania because a person with kleptomania does not realize they have kleptomania. C. The person likely does not have kleptomania because a person with kleptomania is moreconcerned with the act of stealing rather than the obtaining the object which has been stolen. D. The presentation is consistent with kleptomania because the person is stealing things they need.The answer is A- This presentation is consistent with kleptomania
9Question 40 A subcortical dementia with parenchymal abnormalities which can be visualized on MRI is known as which of the following? A. HIV dementia B Kluver-Bucy syndrome. C. Alzheimer’s type dementia D. Creutzfeldt-Jakob disease The answer is C-Alzheimer’s type dementia. Question 41 Predisposing factors for delirium include which of the following? Check all that apply. A. Dementia B. History of falls C. Age 65 and older D. Sensory impairmentThe answers are: A-Dementia, C-Age 65 and older, D-sensory impairment. Step-by-step explanation Question 42 Which of the following diagnostic instruments for Autism Spectrum Disorder is recommended for universal clinical practice? A. Autism Diagnostic Interview -Revised (ADI – R) B. Diagnostic Interview for Social and Communication Disorder (DISCO) C. Development, Dimensional and Diagnostic Interview D. None of the above The answer is A- Autism Diagnostic Interview -Revised (ADI – R) Question 43 A capacity evaluation prior to an organ transplantation is an example of which of the following clinical problems in psychosomatic medicine? A. Psychiatric symptoms secondary to a medical condition B. Medical complications of psychiatric treatments C. Psychological factors precipitating medical conditions D Psychiatric/psychosocial assessment.The answer is D- Psychiatric/psychosocial assessment. Question 44 A temporary marked alteration in the state of consciousness or by the customary sense of personal identity without the replacement by an alternate sense of identity is known as which of the following? A. Ganser Syndrome B. Dissociative Trance Disorder C. Dissociative Identity Disorder D. Factitious Dissociative Identity Disorder The answer is B-Dissociative Trance Disorder
10 Question 45 Which of the following is consistent with current literature about the relationship between obstetrical complications and Autism Spectrum Disorders (ASD)? A. Research is unclear whether obstetric complications are a true risk factor for ASD. B. Research proves there is a negative correlation between obstetrical complications and ASD. C. Research proves there is a positive correlation between obstetrical complications and ASD. D. Research strongly supports a positive relationship between obstetric complications and ASD. The answer is C- Research proves there is a positive correlation between obstetrical complications and ASD Question 46 A major neurocognitive disorder with severe impairment in memory, judgment, orientation, and cognition is known as which of the following? A. Delirium B. Dementia C. Psychosis D. Amnesia The answer is C-Psychosis Question 47 Difficulty with immediate memory is attributed to impairment in which of the following regions of the brain? Select all that apply. A. Broca B. Wernicke C. Occipital D. Temporal The correct answer is D-Temporal lobe. Question 48 Which of the following would not be included in the treatment plan for a patient with illness anxiety disorder? A. Behavioral therapy B. Group psychotherapy C. Insight oriented psychotherapy D. Exploratory invasive procedures to obtain diagnosis The answer is C- Insight oriented psychotherapy Question 49 Which of the following would demonstrate the normal achievement of gross motor developmental milestone for a 6-month-old? A. Pulls to stand B. Transfers items hand to hand
11C. Briefly sits alone: pivots in prone D. Feeds self crackers; stares at new faces. The answer is C- Briefly sits alone: pivots in prone Question 50 A process by which repressed material is brought back to consciousness and the person relives the repressed material accompanied ty the appropriate affective response. A. Abulia. B. Abreaction C. Adynamia D. Alexithymia The answer is B-Abreaction. Question 51 The term psychosomatic literally refers to which of the following? A. Imaginary illness B. Psychiatric illness C. How the mind effects the body D. How the body effects the mindThe answer is A-Imaginary illness. Question 52 Which of the following is consistent with normal range gross motor developmental milestones for a 4-year-old? A. Walks down stairs, jumps backwards B. Balances on one foot for 4 seconds, can broad jump 1 foot C. Writes part of name; copies a square. D. Eats independently, unbuttons items The answer is B- Balances on one foot for 4 seconds, can broad jump 1 foot Question 53 Which of the following speech and language skills are consistent with normal developmental milestones of a 3-year-old? A. Uses three-word sentences; names body parts. B. Uses two-word sentences; understands me and you C. Follows three-step request; tells stories D. Responds to “why?”; likes rhyming words The answer is C- Follows three-step request; tells stories Question 54 Which the following medications is used to treat irritability in autism? A. fluoxetine B. aripiprazole C. alprazolam D. guanfacine The answer is B-aripiprazole
12 Question 55 A congenital neurodevelopmental disorder primarily occurring in females, characterized by specific deficits following a period of normal function growth and development is A. PANDAS B. Rett syndrome C. Reye’s syndrome D. Kluver-Bucy syndrome The answer is B-Ret syndrome Question 56 Which of the following are included in the five different milestone skill areas that should be evaluated? A. Social/emotional skills B. Gross/fine motor skills C. Speech and language skills D. All the above The answer is D-All the above Question 57 Recognizable skills or abilities that have an expected range and order of appearance such as a child taking his first step around the time of his first birthday is known as which of the following? A. Life skills B. Motor developmentC. Developmental history D. Developmental milestones The answer is B-Motor development Question 58 A condition characterized by the person giving approximate answers, with clouding of consciousness, frequently accompanied by hallucinations or other dissociative, somatoform or conversion symptoms isA. Ganser Syndrome B. Schizophrenia C. Dissociative trance disorder D. Dissociative identity disorder The answer is A- Gander Syndrome Question 59 Somatoform disorders represent which type of clinical problem in psychosomatic Medicine? A. Co-occurring medical and psychiatric conditions. B. Psychiatric complications of medical conditions and treatments. C. Psychiatric symptoms secondary to a medical condition. D. Psychological factors precipitating medical symptoms. The answer is D- Psychological factors precipitating medical symptoms.
13 Question 60 Which is NOT consistent with what is known about Gander syndrome? A. Three of Ganser’s first four cases were convicts. B. Cases have been reported in a variety of cultures. C. Women outnumber men by approximately 2 to 1. D. It is considered by some to be an indicator of potential malingering. The answer is B-. Cases have been reported in a variety of cultures Question 61 The second-most common type of dementia caused by cardiovascular and cerebrovascular disease with progressive cognitive decline in stepwise fashion is known as which of the following? A. Pick’s disease B. HIV dementia C. Vascular dementiaD. Lewy-body dementia The answer is C-Vascular dementia. Question 62 Deficits in attention and the ability to complete multi-step commands are associated with impairment in which of the following regions of the brain? A. Frontal B. Prefrontal C. Cingulate gyrus D. All the aboveThe answer is B-Prefrontal cortical area.Question 63: A person erroneously believes they sustained an emotional or physical traumain early life is known as which of the following? A. Ganser syndromeB. False memory syndrome – is characterized by false recollections of a traumatic event, most commonly childhood sexual assault, that the person passionately believes. Adult psychotherapy frequently triggers these pseudo memories, which are frequently vivid and emotionally intense.C. Factitious dissociative identity disorder D. Imitative dissociative identity disorder Question 64: Persons who continually use the internet to play games to the extent that it interferes with social relations and work performance are exhibiting symptoms most specifically consistent with which of the following conditions? A. Obsessive gaming disorder B. Internet gaming disorder – is the excessive use of computers or other devices that allow a user to connect to the Internet, such as tablets and smartphones, for online activitiesto the point where it interferes with daily activities and duties.C. Internet use D. Internet abuse
14Question 65: A new diagnosis in the DSM-5 characterized by persons preoccupied with being sick or developing a disease of some kind is known as which of the following?A. Conversion disorder B. Illness anxiety disorder – The term hypochondriasis was once used to describe illness anxiety disorder. The person is fixated with having or getting sick, and they are continuously concerned about their health.C. Somatic symptom disorder D. Functional neurological symptom disorder Question 66: A subcortical dementia characterized by motor abnormalities including psychomotor slowing, choreo-athetoid movements, executive dysfunction complicated by impaired language, memory, and insight later in the disease process is A. Vascular dementiaB. Huntington’s disease – Huntington’s disease is a neurological disease that is inherited. A symptomatic triad of motor, behavioral, and cognitive changes characterizes it clinically. The majority of Huntington’s disease patients have a choreo-athetoid phenotype, with a lesser proportion having an akinetic/rigid phenotype. Choreic motions tend to lessen as the condition progresses, but akinesia and rigidity become more apparent.C. Creutzfeldt-Jakob diseaseD. Alzheimer’s type dementia Question 67: Which of the following is a common visceral symptom of conversion disorder? A. SeizuresB. DiarrheaC. Paralysis – Conversion disorder is a condition in which a person develops blindness, paralysis, or other nervous system symptoms that are not caused by a physical illness or injury. After a period of mental or physical distress or psychological conflict, symptoms usually appear suddenly.D. Mid-line anesthesia Question 68: The ARNP is working with the family of a patient with Alzheimer’s Disease whokeeps stating the family is plotting against her, trying to have her “snuffed out.” The family is distraught because they state they are doing their best to make sure their family member is safe. The ARNP explains which of the following in educating the patient about the patient’sA. The patient is hallucinating. An estimated 20-30 percent of patient with dementia have hallucinations. – Hallucinations affect 20 to 30 percent of dementia patients (mostly Alzheimer’s patients), and delusions affect 30 to 40 percent of dementia patients, primarily paranoid or persecutory and un systematized delusions, though complex, sustained, and well-systematized delusions are also reported by these patients. In demented people with psychotic symptoms, physical aggression and other forms of violence are widespread.B. The patient is delusional. An estimated 30-40 percent of patients with dementia have delusions. C. The patient likely has valid concerns and adult protective services needs to be called.D. Disturbance in perception is common in patients with Alzheimer’s Disease and patient needs to be hospitalized immediately.
15Question 69: Which of the following questions would be important when differentiating pyromania from schizophrenia?A. Was the fire set in response to a delusion or hallucination?B. Was the fire set deliberately, not a failure to resist an impulse? Pyromania is a condition in which a person’s drive to start fires is compulsive and causes harm to others or their possessions. Pyromania is a pattern of conduct that is deliberate, planned, and repeated.C. Was the fire set as an act of sabotage? D. Was the fire set with a failure to appreciate the consequences of the act? Question 70: The ARNP notices that a patient with a conversion disorder unable to walk hasan inappropriately cavalier attitude toward what seems to be a major impairment and recognizes this an associated psychological symptom known as which of the following?A. IdentificationB. Primary gain C. Secondary gain D. La belle indifference – The paradoxical absence of psychological discomfort despite having a major medical ailment or symptoms related to a health condition is known as la belle indifference, and it is most typically associated with conversion disorder. Question 71: The ARNP recognizes which of the following when the 2-month-old opens her mouth when she sees a bottle. A. The 2-month-old must be quite advanced as this is a visual motor skill normally seen in a 4 month old. B. The 2-month-old is demonstrating a normal developmental adaptive skill. – skills that assist children in becoming more self-reliant.C. The 2-month-old is demonstrating a normal developmental visual motor skill. D. The 2-month-old is demonstrating an advanced developmental gross motor skill. Question 72: Neuropsychiatric testing is defined as which of the following? A. A comprehensive mental status exam B. A neurological evaluation of brain function C. Standardized quantitative reproducible evaluation of a patient’s cognitive abilities. D. A non-invasive test of brain function which analyzes electrical rhythms in the brain – Neuropsychiatric testing is a procedure for determining how well a person’s brain functions. Reading, language use, attention, learning, processing speed, reasoning, memory, problem-solving, mood and personality are among the abilities assessed. Question 73: Deficits in language including naming, repetition, reading and comprehension, and writing is NOT associated with impairment in which of the following regions of the brain? A. BrocaB. Wernicke C. Right parietal – Sensory perception and integration, including taste, hearing, sight, touch, and smell, are all controlled by the parietal lobe. It houses the primary somatic sensory cortex of the brain, which interprets input from other parts of the body.
16D. Left temporal Question 74: A sudden unexpected purposeful travel away from home with inability to recall all of one’s past accompanied by confusion about personal identity not due to direct effects of a substance or a general medical condition which causes clinically significant distress or impairment in social, occupational, or other important areas of functioning is most specifically known as which of the following? A. Dissociative fugue – Dissociative fugue shows that the person is not bewildered or dazed, but rather that he or she appears to be fleeing from something unknown to them. In that both entail the inability to remember key personal information and/or events, it is an element of the dissociative amnesia diagnosis.B. Dissociative amnesia C. Posttraumatic amnesiaD. Systematized amnesia Question 75: Visual hallucinations are associated with impairment in which of the following regions of the brain?A. Occipital B. Temporal C. Left parietal D. Frontal, prefrontal – The impairment of anterior and posterior regions (secondary visualareas, orbitofrontal cortex, and anterior cingulate cortex) involved in a top-down and bottom-up manner, respectively, appears to be linked to visual hallucinations. The most anterior (front) region of the brain is the frontal lobe. It stretches from the back of the skull to the precentral gyrus. Higher cognitive skills such as memory, emotions, impulse control, problem solving, social interaction, and motor function are all controlled by the frontal lobe. Question 76: The principal theoretician to bring psyche and soma together was which of the following? A. Sigmund Freud – According to Sigmund Freud, a famous psychotherapist, children go through a succession of psychosexual stages before developing an adult personality. His hypothesis explained how personality evolved during childhood.B. Anna Freud C. Karl Abraham D. Georg Grodecki Question 77: Which of the following medications are FDA-approved medications for the treatment of delirium? A. Donepezil B. GalantamineC. Rivastigmine D. None of the above – Antipsychotics are routinely utilized as a first-line pharmacologic method to control symptoms that risk safety or impede care when Nonpharmacological techniques are ineffective, despite the fact that there are no FDA-approved drugs for the treatment of delirium.
17Question 78: Anxiety related to chemotherapy is an example of which type of clinical problem in psychosomatic medicine? A. Psychological factors precipitating medical symptoms – Medical symptoms are exacerbated by psychological issues. Certain psychological issues, such as sadness or anxiety, stressful life experiences, differences in relationship styles, personality traits, and coping techniques, might have a negative impact on medical disorders. Psychological factors affecting other medical conditions is a disorder that occurs when psychological or behavioral factors have an adverse effect on a general medical condition, the factors may precipitate or exacerbate the medical condition, interfere with treatment, or contribute to morbidity and mortality.B. Psychiatric symptoms as a reaction to a medical condition or treatment C. Co-occurring medical and psychiatric conditions D. Psychiatric complications of medical conditions and treatments Question 79: A type of delirium characterized by cycling through psychomotor agitation and retardation, from apathy to hypervigilance is known as which of the following? A. Mixed delirium B. Bipolar delirium C. Hyperactive delirium – Increased motor activity, restlessness, agitation, hostility, wandering, hyper alertness, hallucinations and delusions, and inappropriate behavior are all symptoms of hyperactive delirium.D. A and B Question 80: A tension state that can exist without an action is known as which of the following? A. An obsession B. A compulsionC. An impulse – An impulse is a sudden force or desire, such as an electrical impulse or anurge. You’re following an impulse if you act on a sudden feeling or thinking. That’s similar to a whim, it’s not something you’ve given much consideration to.D. Ego dystonicQuestion 81 Which of the following is true regarding how intermittent explosive disorder is differentiated from antisocial or borderline personality disorder? D. In personality disorders, aggressiveness and impulsivity is part of the person’s character and is present between outbursts. Intermittent Explosive Disorder (IED) is an impulse control disorder characterized by aggressive outbursts that has a rapid onset and, typically, with little to no warning Question 82 A tension state that always has an action component is known as which of the following? C. A compulsion Compulsions are behaviors an individual engages in to attempt to get rid of the obsessions and/or decrease his or her distress.
18Question 83 A cognitive assessment should include which of the following?D. All of the above Question 84 The ARNP evaluates a 4-year-old who cannot balance on one foot for 3 seconds, cannot copy a circle and realizes which of the following? D. This is a minor concern, the APRN advises to enroll the child in gymnastics for balance and an art class to learn to draw better. Question 85 Which of the following adaptive skills are consist with normal range developmental milestone of an 18-month-old? B. Finger feeds items; takes off a hat. 18-month-old toddler at this age, children love to play and explore. They begin to show some independence and may play pretend and point at objects they want. They also begin to understand what things in the house are used for, such as a cup or spoon. Question 86 A disorder characterized by the presence of two or more distinct identities which differ from each other in that each presents as having its own pattern of perceiving, relating to, and thinking about the environment and self. A. Dissociative Identity Disorder Dissociative identity disorder is characterized by the presence of two or more distinct or split identities or personality states that continually have power over the person’s behavior. Question 87 Interest in fires and deliberate setting of fires for financial gain is known as which of the following?B. Pyromania Pyromania is a type of impulse control disorder that is characterized by being unable to resist starting fires. People with pyromania know that setting fires is harmful.But setting fires is the only way they can relieve their built-up tension, anxiety, or arousal Question 88 Which of the following conversion disorder symptoms are associated with a good prognosis? A. Tremors and aphonia Symptoms of conversion disorder usually last for days to weeks and may suddenly go away. Usually the symptom itself is not life-threatening, but complications of the symptoms or unnecessary medical tests can be debilitating. For most people, symptoms of conversion disorder get better with reassurance and time.
19 Question 89A type of delirium characterized by psychomotor retardation and apathy is known as which of the following? C. Hypoactive delirium Hypoactive delirium presents with lethargy, drowsiness, apathy, decreased responsiveness, or slowed motor skills. In mixed delirium, individuals display either relatively normal levels of psychomotor activity or rapidly fluctuating levels of activity. Question 90 Which of the following symptoms assist in differentiating a seizure from a pseudo seizure? A. Tongue biting is typically not present in a pseudoseizure B. Urinary incontinence is typically not present in a pseudoseizure C. Injuries from falling are typically not present in a pseudoseizure D. All the above Question 91 Which of the following questions are recommended to assess sexual identity in a male adolescent?A. Do you have or have you had a girlfriend? B. Do you have or have you had a boyfriend? C. Do you have, or have you had romantic feelings toward anyone? D. A and C only Question 92 Which of the following demographics are consistent with autism spectrum disorder (ASD)? A. Female to male ratio (4:1) B. Onset of symptoms by age 3 C. Affects 1 in 100 children D. All the above Question 93 The diagnosis formerly known as multiple personality disorder is now known as which of thefollowing? B. Dissociative identity disorder Dissociative identity disorder was previously referred toas multiple personality disorder. Symptoms of dissociative identity disorder (criteria for diagnosis) include: The existence of two or more distinct identities (or “personality states”). Question 94 Which of the following is NOT consistent with what is known about intermittent explosive disorder across the lifespan?
20A. Intermittent explosive disorder may appear at any stage of life. This diagnosis is not given prior to the age of 6 years and the symptoms typically first appear in late childhood or adolescenceB. Intermittent explosive disorder usually appears between late adolescence and early adulthood. C. Intermittent explosive disorder typically increases in severity with the onset of middle age. D. The onset of intermittent explosive disorder may be acute or insidious Question 95 Functional neurological symptoms disorder is also known as which of the following? A. Conversion disorder A conversion disorder, also called Disorder “Functional Neurological Symptom Disorder” is a relatively uncommon mental disorder. Typically, the person has physical symptoms that no medical condition, physical examination or testing can explain Question 96 Which of the following would be a developmental trigger for a 3-year-old? Check all that apply. A. Cannot balance on one foot for 3 seconds, cannot copy a circle B. Lack of imaginative plan; cannot hypothesize an other’s thought C. Cannot use a three-word sentence; speech only 50% understandable D. Speech less than 75% understandable; cannot identify self or details in pictures Question 97 Which of the following questions would be important when differentiating pyromania from conduct disorder or antisocial personality disorder? Check all that apply. A. Was the fire set in response to a delusion or hallucination? B. Was the fire set deliberately, not a failure to resist an impulse? C. Was the fire set as an act of sabotage? D. Was the fire set with a failure to appreciate the consequences of the act? Question 98 The therapeutic approach that has the best chance of success for the person with intermittent explosive disorder is which of the following? A. Psychotherapy only B. Psychopharmacology only C. A combined psychotherapy/psychopharmacology approach D. Family therapy only Question 99
21Which of the following are common disorders that must be differentiated from dissociative identity disorder? Check all that apply. A. Perimenstrual disorders B. Posttraumatic stress disorder C. Obsessive-compulsive disorder D B and C only. Question 100 Which of the following is an example of a medical complication of psychiatric conditions or treatment? A. Dementia B. Neuroleptic Malignant syndrome Neuroleptic malignant syndrome (NMS) is a life-threatening idiosyncratic reaction to antipsychotic drugs characterized by fever, altered mental status, muscle rigidity, and autonomic dysfunction.C. Depression related to limb amputationD. Recurrence of depressive disorder in setting of cancer treatment

Leave a Comment

Scroll to Top